Friday, September 6, 2013

  1. Must be true/Inference/Conclusion CR: You have to consider whatever stated in the stimulus as true.
Correct choices can be:
    1. Paraphrased of some part or entire stimulus
    2. Combination of two or more premises

Incorrect choices can be:
a.       Could be true or Likely to be true choices
b.      Exaggerated/Extreme case answers
c.       New information (not explicitly mentioned in the stimulus) containing choices
d.      Shell game (an idea, in one of the choices, similar to the one mentioned in the stimulus, but in a modified form to make it attractive choice. This will be a Trap)
e.       Opposite answer (opposite to the stated facts in the stimulus). For example, stimulus shows case >>> effect while the choice says effect >>> cause.
f.       Reverse order (rearrangement of some term in the answer choices which are also mentioned in the stimulus). For example,
Many politicians are having some type of security gadgets.

While the choice says:
Some politicians are having many types of security gadgets.

  1. Main Point Question: It is same as the Must be True/Inference types. You have to consider whatever stated in the stimulus as true but the correct choice reflects the author’s point of view and all facts in this choice are also reflected in the stimulus.

            Incorrect choices can be:
a.       Choices that repeats one/some of the premise(s) of the argument.
b.      Answers that are true as per the argument but do not capture author’s point.
c.       Exaggerated/Extreme case answers
g.      New information (not explicitly mentioned in the stimulus) containing choices
d.      Shell game
e.       Choices mentioning the opposite answer
f.       Choices in Reverse order

  1. Weaken the argument/conclusion: The best part of this type of question is availability of conclusion in the argument. If you can spot the conclusion then your chances to select the best among weakening choices will be higher, because you may find several choices may be weakening the conclusion but there will be one which is weakening the MOST.

Remember, the following:
a.       You need to question the validity of the argument not the answer choices.

b.      In the question stem you will find words indicating weaken the conclusion:

Weaken, attack, undermine, contradicts, evidence against, challenge, damage, counter, refute, argue against, cast doubt, criticize, call into question, etc.

c.       There are traces of flaw/error in the reasoning given in the argument. Read closely to spot them. This error can help you to select the best choice.

d.      Even if the choice introduces new information, you need to accept them as correct choices, if they are weakening the conclusion.

e.       We need to call into question the conclusion by showing that the author fails to address some crucial part/information to arrive at the conclusion. This missed part is assumption on which the conclusion is based. We are not to destroy the conclusion, though it can be a correct choice.

f.       Try to personalize the choices as if these are asked to you. What would you answer for them? This will become easier once you start practice.

Example: All software engineers go to US. Mr. X went to US. Therefore, Mr. X is a software engineer.

Not necessarily. This is just oversimplified statement. Mr. X can be a child care activist, who met the senator in US for some funds. If the argument were,

Only software engineers go to US. Mr. X went to US. Therefore, Mr. X is a software engineer.

True.

Correct choices can be identified by:
    1. Choices indicating incomplete information (new info), which is missed by the author as one of the possibility.
    2. Improper comparison b/w two things which are altogether different.

Incorrect answer choices: I call them Fallacies/Traps.
a.       Opposite answer: This is almost present as one of the answer choices. Refer the Must be True question section for some details. This choice will lead you to think in the opposite direction w.r.t. the conclusion.
b.      Shell game choice: Refer the Must be True question section for some details. Remember, in shell game, a similar reasoning/idea is presented similar to one of the premises but it is in a modified form.
c.       Out of scope choice: choices which are not as per the context. For example, the argument is on plant in US while this choice is on plants in other countries.
Now, lets take an example from OG10 to analyze different types of incorrect weakening choices:

110. Robot satellites relay important communications and identify weather patterns. Because the satellites can be repaired only in orbit, astronauts are needed to repair them. Without repairs, the satellites would eventually malfunction. Therefore, space flights carrying astronauts must continue.

Which of the following, if true, would most seriously weaken the argument above?
(A) Satellites falling from orbit because of malfunctions burn up in the atmosphere.
(B) Although satellites are indispensable in the identification of weather patterns, weather forecasters also make some use of computer projections to identify weather patters.
(C) The government, responding to public pressure, has decided to cut the budget for space flights and put more money into social welfare programs.
(D) Repair of satellites requires heavy equipment, which adds to the amount of fuel needed to lift a spaceship carrying astronauts into orbit.
(E) Technical obsolescence of robot satellites makes repairing them more costly and less practical than sending new, improved satellites into orbit.


Conclusion: Therefore, space flights carrying astronauts must continue.
A choice should call into this conclusion as - space flights carrying astronauts must NOT continue.

First thing you need to understand why space flights carrying astronauts must continue. Is it because the satellites can be repaired only in orbit by astronauts? Is there any other way the satellites can be repaired (by robots, technologies such as remote sensing from Earth, etc) or send new and efficient satellites which are more economical to sending satellites carrying astronauts.

(A) Satellites falling from orbit because of malfunctions burn up in the atmosphere. [Out of scope choice. Although new info but irrelevant to our case. Incorrect]
(B) Although satellites are indispensable in the identification of weather patterns, weather forecasters also make some use of computer projections to identify weather patters. [Some details, in fact, are mentioned in the argument on weather patterns but nothing on the use of computer projections. This is indeed new info but it is out of scope to our concern. Incorrect]
(C) The government, responding to public pressure, has decided to cut the budget for space flights and put more money into social welfare programs. [All underlined parts are new info and irrelevant to this case. Incorrect]
(D) Repair of satellites requires heavy equipment, which adds to the amount of fuel needed to lift a spaceship carrying astronauts into orbit. [Beware, this is Shell Game fallacy. Both underlined parts are mentioned in the argument but there is some modification in their connectivity to mislead the test taker. Incorrect]
(E) Technical obsolescence of robot satellites makes repairing them more costly and less practical than sending new, improved satellites into orbit. [This is what missing as one of the possibilities against sending satellites carrying astronauts. Correct]

  1. Cause-Effect Reasoning
  2. Strengthen and Assumption: Most challenging and frequently occurring types of questions on GMAT.

Remember the following:
    1. There will always be one main conclusion present either in argument or in the question stem.
    2. You need to understand the organization of the argument, premises and conclusion, in order to find the correct answer choices.
    3. Read the stimulus carefully as there can be reasoning errors/gaps, which are crucial for assumption based questions.
    4. Even if the choice introduces new information, you need to accept them as correct choices, if they are fitting in the context for the conclusion.
    5. Assumptions are unstated premises, which connect the premises/facts with the main conclusion, while the Strengthen part provides additional support to the conclusion. You can say Strengthen choices advocate the conclusion.
    6. This is most critical-You need to consider all the answer choices as correct and the stimulus is in question. Only the metal each choice carry varies. I mean these all may support the argument from 1%-100% but there is only one choice which is strengthening the argument the MOST.

Strengthen types:
a. The question stem will have indicators such as:

strengthen, support, helps, justifies

Correct choices:
a.       Identify the conclusion and connect the answer choices with it to answer the question – does this choice helps the author? Is yes, this is your answer.
b.      While understanding the organization of the argument, if you find weaknesses/flaws/gaps, which are made intentionally, and one of the answer choices is removing this problem then its your answer.
c.       If there is conclusion in the question stem then there are good chances of finding the:
1.      EXCEPT/LEAST Strengthen choices
2.      A missing info (assumption) is asked to support the conclusion.

Incorrect choices:
a.       Opposite answers: These answers are just the opposite of strengthening the conclusion, i.e., these weaken the conclusion.
b.      Shell Game answers: This type of answer support a conclusion that is similar to, but slightly different from, the one present in the stimulus. In other words, this choice will have a similar but slightly different conclusion.
c.       Out of scope: These are the most common and more in number among the 5 choices. These bring irrelevant or narrow scope information to the argument.
d.      Paraphrased premise: One of the premises is represented in a paraphrased form just to mislead.

Now, take an example from OG10 to analyze different types of strengthening choices:

60. Since the routine use of antibiotics can give rise to resistant bacteria capable of surviving antibiotic environments, the presence of resistant bacteria in people could be due to the human use of prescription antibiotics. Some scientists, however, believe that most resistant bacteria in people derive from human consumption of bacterially infected meat.

Which of the following statements, if true, would most significantly strengthen the hypothesis of the scientists?

(A) Antibiotics are routinely included in livestock feed so that livestock producers can increase the rate of growth of their animals.
(B) Most people who develop food poisoning from bacterially infected meat are treated with prescription antibiotics.
(C) The incidence of resistant bacteria in people has tended to be much higher in urban areas than in rural areas where meat is of comparable quality.
(D) People who have never taken prescription antibiotics are those least likely to develop resistant bacteria.
(E) Livestock producers claim that resistant bacteria in animals cannot be transmitted to people through infected meat.

Premises: The routine use of antibiotics can give rise to resistant bacteria capable of surviving antibiotic environments.

Sub-conclusion: The presence of resistant bacteria in people could be due to the human use of prescription antibiotics.

Counter Conclusion: Some scientists, however, believe that most resistant
bacteria in people derive from human consumption of bacterially infected meat.

In the question stem, the author asked for strengthening the scientists’ hypothesis.
So, this question type is our category c of Correct choices section (refer above).
We are asked for the missing link to prove the hypothesis as the correct
conclusion. Now, let’s check individual answer choices:
(A) Antibiotics are routinely included in livestock feed so that livestock producers can increase the rate of growth of their animals. [This is the missing link. Correct]
(B) Most people who develop food poisoning from bacterially infected meat are treated with prescription antibiotics. [New information, which is Out of scope answer. Incorrect]
(C) The incidence of resistant bacteria in people has tended to be much higher in urban areas than in rural areas where meat is of comparable quality. [Out of scope answer. Incorrect]
(D) People who have never taken prescription antibiotics are those least likely to develop resistant bacteria. [This is a Shell Game trap. This option is in negation to above premises and very narrow in scope. Incorrect]
(E) Livestock producers claim that resistant bacteria in animals cannot be
transmitted to people through infected meat. [It is a case of Opposite answer. This
is just stating against the scientists’ hypothesis. Incorrect]


Assumption based questions: Assumption based questions are usually the most
challenging and trickiest questions on GMAT.

Remember the following (Most of the following points are same as of the Strengthen types):
a.       There will always be one main conclusion present either in argument or in the question stem.

b.      You need to understand the organization of the argument, premises and conclusion, in order to find the correct answer choices.

c.       Read the stimulus carefully as there can be reasoning errors/gaps, which are crucial for assumption based questions.

d.      Even if the choice introduces new information, you need to accept them as correct choices, if they are fitting in the context for the conclusion. Remember, if the new information is present in the answer choices then to qualify for a correct contender, this info should be a subset of the main statement in the stimulus. For example – Men (main statement): White men, Black men, small men, etc (subsets of Men).

e.       Assumptions are unstated premises, which connect the premises/facts with the main conclusion. This is the most important part of the stimulus. Thus, assumption will work like either a Supporter or a Defender:

Premise 1, premise 2, assumption, premise 3, conclusion

In this case, it is supporting the premises to lead to the conclusion.


Premise 1, premise 2, premise 3, assumption, conclusion

In this case, it is supported by the premises to lead to the conclusion.

Often, new information is added as assumption in such cases. If you find an answer choice that contain new info, stop and check whether this new info is in context and filling some gaps in the argument. If Yes, then that is your correct answer.

The author reasoning must be air-tight and every possible objection has been considered and rejected. It means the idea that can weaken the conclusion is already addressed in the stimulus or cannot occur. Here comes the role of Defending Assumption types. If you do not find gaps in the argument then this indicates that you can expect a Defender based assumption question. For example:

People who read a lot are more intelligent than the other people. Thus, reading must cause a person to be intelligent

Conclusion: Reading must cause a person to be intelligent.

So, to defend the argument, the author must have assumed that intelligence is solely based on reading and cannot happen by other factors such as sleeping more, regular exercise, a good diet or genetics, which, if proved, can weaken the conclusion.

So, Defender assumptions eliminate a possible source of attack on conclusion.

f.       This is most critical-You need to consider all the answer choices as correct and the stimulus is in question. Only the metal each choice carry varies. The conclusion must pass the correct assumption test else it will fall apart.

g.      Assumption question stem contains following types of indicating words:
·         Author assumes,
·         conclusion can’t be true unless which of the following is also true
·         validity of the conclusion is checked by
·         argument above depends on which of the following
Steps to find the correct/incorrect assumption answers choices:
a.       Logically negate the answer choice, which you feel is the assumption (don’t apply this rule to all 5 choices as this will be time consuming). If by logically negating the choice, the argument and the conclusion falls apart (in simpler terms, if negated answer choices weakens the conclusion), then this is your correct choice. Remember, only one option, upon negation, can decide the validity of argument/conclusion.

b.      If an answer choice starts with “At least one” or “At least some” then the chances are high for this option to be the correct one. But, do not simply assume that this is the only one. Try to negate and check the argument.

c.       Shell Game Trap. These will usually be present as one of the option.

d.      If any answer choice mentions extreme/strong reasons such as:
The most
Main factor/objective
Primary reason

Usually these answers fall in the incorrect choice category.

e.       Watch out for negative indictors (not, never, no). These can be
your Defender assumption answer choice.

f.       Cause-Effect relationship in Assumptions:
When the author mentions a certain cause-effect relation then it is assumed that it is true. The stated cause is the only cause which will always produce the effect. So, correct answer choice will fall into one of the following:
g.      Eliminate an alternate cause for the stated effect in the argument.
h.      Shows that when the cause occurs, the effect occurs.
i.        Shows when the cause does not occur, the effect does not occur.
j.        Eliminates the reversed relationship (effect >>> cause). This type is one of the most frequently occurring types.
k.      Shows that the data used to make the casual statement are accurate, or eliminate possible problems with the data.

Now, let’s analyze one CR question from OG10:

48. A researcher discovered that people who have low levels of immune-system activity tend to score much lower on tests of mental health than do people with normal or high immune-system activity. The researcher concluded from this experiment that the immune system protects against mental illness as well as against physical disease.

The researcher’s conclusion depends on which of the following assumptions?
A. High immune-system activity protects against mental illness better than normal immune-system activity does.
B. Mental illness is similar to physical disease in its effects on body systems.
C. People with high immune-system activity cannot develop mental illness.
D. Mental illness does not cause people’s immune-system activity to decrease.
E. Psychological treatment of mental illness is not as effective as is medical treatment.

Premise 1: A researcher discovered that, on tests of mental health, people who have low levels of immune-system activity tend to score much < people with normal or high immune-system activity.

Conclusion: Immune system protects against mental illness as well as against physical disease.

In the premises, we are given some details on mental health tests, low levels of immune system activity, normal or high immune-system activity and score while in the conclusion, we are given Immune system, protection, mental illness and physical disease. We are provided with info on MENTAL TESTS and the direct relationship b/w them:

The higher the level of immune-system activity >>> the higher score you will get

But wait, no details on mental illness in the premises. The researcher forgot to mention the relationship b/w immune system and mental-illness. Also, this is the catch to decide that we will have Defender assumption.

Now, let’s break and analyze the answer choices:

A. High immune-system activity protects against mental illness better than normal immune-system activity does. [There is no casual relationship b/w mental illness and immune-system activity mentioned in the conclusion. Incorrect]

B. Mental illness is similar to physical disease in its effects on body systems. [This is a case of Shell Game fallacy. In the conclusion statement, mental illness and physical decease are mentioned and thus this choice is created to mislead. But, it fails to connect with mental illness. Incorrect]

C. People with high immune-system activity cannot develop mental illness. [There is no casual relationship b/w mental illness and immune-system activity mentioned in the conclusion. This is adding extreme case and also, this is OPPOSITE to the conclusion. Incorrect]

D. Mental illness does not cause people’s immune-system activity to decrease. [If mental illness doesn’t change the immune-system activity then this is the case which author missed to defend the argument. Correct]

E. Psychological treatment of mental illness is not as effective as is medical treatment. [This is completely out of scope new information. No relationship b/w treatment and illness is in context of the conclusion. Incorrect]